Complex Eigenvalues and Eigenvectors of a 2x2 Matrix: Diagonalizable or Not?

  • Thread starter Thread starter Iconate
  • Start date Start date
  • Tags Tags
    Complex
Click For Summary
The discussion focuses on determining the eigenvalues and eigenvectors of the matrix A = [0 -i | i 0] and whether it is diagonalizable. The initial claim was that A cannot be diagonalized due to the characteristic polynomial yielding complex eigenvalues λ1 = -i and λ2 = i, leading to a trivial solution. However, upon reevaluation, the correct characteristic polynomial is λ^2 - 1 = 0, resulting in real eigenvalues λ1 = 1 and λ2 = -1. This correction indicates that the matrix A is indeed diagonalizable, and the discussion concludes with the need to find the matrices P and D for the diagonalization.
Iconate
Messages
20
Reaction score
0

Homework Statement


Find the eigenvalues and eigenvectors of A. (Both eigenvalues and eigenvectors are now allowed
to be complex.) Is it diagonalizable? Explain why or why not. If it is diagonalizable, explicitly
find matrices P and D such that
A = PDP−1
where D is a diagonal 2 × 2 matrix.

A = [ 0 -i | i 0 ]

The Attempt at a Solution



I determined that A cannot be diagonalized because, by the characteristic polynomial equation we get \lambda2 + 1 = 0

Therefore \lambda1 = -i \lambda2 = i

plugging \lambda2 into my matrix A I get:

ix + iy = 0
-ix + iy = 0

but the only solution to this is x=y=0, I get the same result for \lambda1

Is this correct? I have a feeling this trivial solution is wrong
I tried row reduction, but I still get the same result.
 
Physics news on Phys.org
so many minuses!

Hi Iconate! :smile:

(have a lambda: λ :wink:)
Iconate said:
A = [ 0 -i | i 0 ]

… by the characteristic polynomial equation we get \lambda2 + 1 = 0

Noooo! :redface:
 


Ahhh I see
my determinant should be
λ2 - (-i)(i) = 0
λ2 + (i2) = 0
λ2 - 1 = 0

thus λ1 = 1 λ1 = -1

Thanks >.<
 
Question: A clock's minute hand has length 4 and its hour hand has length 3. What is the distance between the tips at the moment when it is increasing most rapidly?(Putnam Exam Question) Answer: Making assumption that both the hands moves at constant angular velocities, the answer is ## \sqrt{7} .## But don't you think this assumption is somewhat doubtful and wrong?

Similar threads

  • · Replies 6 ·
Replies
6
Views
2K
  • · Replies 7 ·
Replies
7
Views
2K
Replies
5
Views
2K
  • · Replies 2 ·
Replies
2
Views
3K
  • · Replies 2 ·
Replies
2
Views
1K
  • · Replies 8 ·
Replies
8
Views
2K
  • · Replies 4 ·
Replies
4
Views
1K
  • · Replies 3 ·
Replies
3
Views
2K
  • · Replies 2 ·
Replies
2
Views
2K
  • · Replies 5 ·
Replies
5
Views
2K